免費論壇 繁體 | 簡體
Sclub交友聊天~加入聊天室當版主
分享
返回列表 发帖

最近在知乎的回答存档——其他类

前往→前言
前往→不等式类
前往→几何类


网址:https://www.zhihu.com/question/430131586
标题:这个极限怎么计算?;?
v2-f83fddcd1d7d87fed82256fa113a8ddf_r.jpg
2021-7-19 22:53

我的回答:
因为
\begin{align*} &(1+x)(1+x^2)(1+x^4)(1+x^8)\cdots (1+x^{2^n})\\ ={}&\frac {(1-x^2)(1+x^2)(1+x^4)(1+x^8)\cdots (1+x^{2^n})}{1-x}\\ ={}&\frac {(1-x^4)(1+x^4)(1+x^8)\cdots (1+x^{2^n})}{1-x}\\ ={}&\frac {(1-x^8)(1+x^8)\cdots (1+x^{2^n})}{1-x}\\ ={}&\cdots \\ ={}&\frac {1-x^{2^{n+1}}}{1-x}, \end{align*}令 `x=\frac12` 即得
\[\frac 32\cdot \frac 54\cdot \frac {17}{16}\cdot \frac {257}{256}\cdots \frac {2^{2^n}+1}{2^{2^n}}=2\left( {1-\frac 1{2^{2^{n+1}}}} \right),\]故所求极限为 2。

发布于 01-16
分享到: QQ空间QQ空间 腾讯微博腾讯微博 腾讯朋友腾讯朋友
$\href{https://kuingggg.github.io/}{\text{About Me}}$

网址:https://www.zhihu.com/question/422786851
标题:xcothx≤1-2ln(x/sinhx)怎么证明?
有没有除了泰勒展开以外的,比较漂亮的解答(*°∀°)=3

我的回答:
欣赏 @虚调子 的精彩证法之余,补充一个普通的导数证法:
令 `f(x)=\text{右边}-\text{左边}` ,显然 `f(x)` 是偶函数,只需考虑 `x>0` 的情况,又 `x\to 0` 时 `f(x)\to 0` ,故只需证明 `f'(x)>0` ,求导计算可得
\[f'(x)=\frac {x^2+x\sinh x\cosh x-2\sinh ^2x}{x\sinh ^2x},\]令
\[g(x)=x^2+x\sinh x\cosh x-2\sinh ^2x=x^2+\frac x2\sinh 2x-\cosh 2x+1,\]只需证明 `g(x)>0` ,继续求导得
\begin{align*} g'(x)&=2x+x\cosh 2x-\frac 32\sinh 2x,\\ g''(x)&=2+2x\sinh 2x-2\cosh 2x=2\sinh 2x(x-\tanh x)>0, \end{align*}所以 `g'(x)>g'(0)=0` ,进而 `g(x)>g(0)=0` ,即得证。

发布于 01-17

注:网友“虚调子”的证法可以说是看穿了这个不等式,推荐大家看看。

TOP

网址:https://www.zhihu.com/question/441111257
标题:请问三边长均为整数且周长为 n 的三角形共有多少种不同的形状?
注:不同的形状是指不同的三边长(没有顺序)。
如边长为 3,4,5 与边长为 5,4,3 即视作一种形状。

我的回答:
谢谢网友 @David KZ 指出,之前的解答忘记考虑得三角形,需要 a+b>c,还好修正不需要大改。

设三边 `a\leqslant b\leqslant c` ,就是求 `a+b+c=n` 且 `a+b>c` 的解数。

令 `a=1+x,b=1+x+y,c=1+x+y+z` ,变为 `3x+2y+z=n-3` 且 `1+x>z` ,即 `x\geqslant z` ,于是再设 `x=z+w` ,问题就变成:

求 `3w+2y+4z=n-3` 的非负整数解数。

令 `F=(1+x^3+x^6+\cdots )(1+x^2+x^4+\cdots )(1+x^4+x^8+\cdots )` ,变成求 F 的展开式中的 `x^{n-3}` 的系数,而
\[F=\frac 1{(1-x^3)(1-x^2)(1-x^4)}=\frac M{(1-x^{12})^3}=M\sum_{k=0}^\infty C_{k+2}^2x^{12k},\]其中
\begin{align*} M={}&(1+x^3+x^6+x^9)(1+x^2+x^4+\cdots +x^{10})(1+x^4+x^8)\\ ={}&1+x^2+x^3+2x^4+x^5+3x^6+2x^7+4x^8+3x^9+5x^{10}+4x^{11}\\ &+4x^{12}+5x^{13}+5x^{14}+4x^{15}+4x^{16}+5x^{17}+3x^{18}+4x^{19}+2x^{20}+3x^{21}+x^{22}+2x^{23}\\ &+x^{24}+x^{25}+x^{27}, \end{align*}这样就可以计算具体的数字了。

比如 n=102,即求 F 的 `x^{99}` 的系数,由于 `99=8\times12+3=7\times12+15=6\times12+27` ,而 M 中有 `x^3,4x^{15},x^{27}` 这三项,故此 `x^{99}` 的系数就是 `C_{8+2}^2+4C_{7+2}^2+C_{6+2}^2=217` 。

编辑于 01-25

注:还是用的之前的招……
其他回答里的通项式有空也验证一下,另附 oeis 的链接:http://oeis.org/A005044

TOP

网址:https://www.zhihu.com/question/443436933
标题:怎么巧妙地解这道题?
v2-afa4cf30744badd0d49e01b4a381e3c6_r.jpg
2021-7-20 14:28

我的回答:
\[x^2+y^2+e^y\left( \frac {e^y}2-x-y \right)=\left( x-\frac {e^y}2 \right)^2+\left( y-\frac {e^y}2 \right)^2\geqslant \left( y-\frac {e^y}2 \right)^2,\]令 `f(y)=y-\frac {e^y}2` ,有 `f'(y)=1-\frac {e^y}2` ,故 `f(y)\leqslant f(\ln 2)=\ln 2-1` ,所以 `f(y)^2\geqslant (1-\ln 2)^2` ,即
\[x^2+y^2+e^y\left( \frac {e^y}2-x-y \right)\geqslant (1-\ln 2)^2,\]当 `x=1,y=\ln 2` 时取等。

发布于 02-08

TOP

网址:https://www.zhihu.com/question/443748756
标题:如何证明下面的函数在(0,π/2)上不小于0?
`2cosx+sin2x-\sqrt{3}sin3x-\sqrt{3}/2`
做题时遇到这种三角复合函数总是不知道怎么处理,求导也只是在转圈圈……
求教

我的回答:
记 `F=2\cos x+\sin2x-\sqrt3\sin3x-\frac{\sqrt3}2` ,有
\begin{align*} \frac{\sqrt3}2F&=(5\sin x+4)\left( \sin x-\frac12 \right)^2-(\sin x+1)\left( \cos x-\frac{\sqrt3}2 \right)^2\\ &\geqslant(\sin x+1)\left( 4\left( \sin x-\frac12 \right)^2-\left( \cos x-\frac{\sqrt3}2 \right)^2 \right), \end{align*}故只需证
\[2\left| \sin x-\frac12 \right|\geqslant\left| \cos x-\frac{\sqrt3}2 \right|,\]当 `x=30^\circ` 时两边为零,当 `x\ne30^\circ` 时上式即
\[\left| \frac{\sin x-\sin30^\circ}{\cos x-\cos30^\circ} \right|\geqslant\frac12\iff\cot\frac{x+30^\circ}2\geqslant\frac12,\]显然有 `LHS>\cot 60^\circ =\frac 1{\sqrt 3}>\frac 12` ,综上即得 `F\geqslant0` 。

发布于 02-11

注:原帖有其他解法也可以看看。

TOP

网址:https://www.zhihu.com/question/452160528
标题:这两个恒等式如何证明?
v2-a84957804e3dd6a6599aaadf7fc6182b_b.jpeg
2021-7-20 23:57

我的回答:
令 `f(t)=(1+at)^n=\sum_{j=0}^n\binom nja^jt^j` ,求导得
\begin{align*} f'(t)&=n(1+at)^{n-1}a=\sum_{j=0}^nj\binom nja^jt^{j-1},\\ f''(t)&=n(n-1)(1+at)^{n-2}a^2=\sum_{j=0}^nj(j-1)\binom nja^jt^{j-2},\\ f'''(t)&=n(n-1)(n-2)(1+at)^{n-3}a^3=\sum_{j=0}^nj(j-1)(j-2)\binom nja^jt^{j-3},\\ &\cdots \end{align*}令 t=1 得
\begin{align*} \sum_{j=0}^nj\binom nja^j&=n(1+a)^n\frac a{1+a},\\ \sum_{j=0}^nj(j-1)\binom nja^j&=n(n-1)(1+a)^n\left( \frac a{1+a} \right)^2,\\ \sum_{j=0}^nj(j-1)(j-2)\binom nja^j&=n(n-1)(n-2)(1+a)^n\left( \frac a{1+a} \right)^3,\\ &\cdots \end{align*}令 `a=\frac x{1-x}` ,则 `1+a=\frac 1{1-x}` 且 `\frac a{1+a}=x` ,故
\begin{align*} \sum_{j=0}^nj\binom nj\left( {\frac x{1-x}} \right)^j&=\frac {nx}{(1-x)^n},\\ \sum_{j=0}^nj(j-1)\binom nj\left( \frac x{1-x} \right)^j&=\frac {n(n-1)x^2}{(1-x)^n},\\ \sum_{j=0}^nj(j-1)(j-2)\binom nj\left( \frac x{1-x} \right)^j&=\frac {n(n-1)(n-2)x^3}{(1-x)^n},\\ &\cdots \end{align*}去分母之后,第一式,以及前两式相加,即得
\begin{align*} \sum_{j=0}^nj\binom njx^j(1-x)^{n-j}&=nx,\\ \sum_{j=0}^nj^2\binom njx^j(1-x)^{n-j}&=n(n-1)x^2+nx, \end{align*}顺便算个立方,由 `j^3=j(j-1)(j-2)+3j(j-1)+j` 即得
\[\sum_{j=0}^nj^3\binom njx^j(1-x)^{n-j}=n(n-1)(n-2)x^3+3n(n-1)x^2+nx,\]更高次方也可以继续算下去,如`j^4=j(j-1)(j-2)(j-3)+6j(j-1)(j-2)+7j(j-1)+j` 于是四次方的那几个系数就是 {1,6,7,1},下一个五次方是 {1,10,25,15,1},规律是:若 k 次方的系数列是 `\{a_1,a_2,a_3,\ldots ,a_{k-1},a_k\}` 则 k+1 次的是 `\{a_1,ka_1+a_2,(k-1)a_2+a_3,\ldots ,3a_{k-2}+a_{k-1},2a_{k-1}+a_k,a_k\}` 。

发布于 03-31

注:其实还是当年这帖的方法了,战巡的方法我没写上,除了因为懒,还有就是虽然用 `(1+e^x)^n` 的 m 阶导数表示的形式最简单,但如果没有计算机,计算量还是不小的,而我的系数生成列更容易手算。

TOP

网址:https://www.zhihu.com/question/454481092
标题:函数 f(x)=x/2+√(x²-x+1) 的最小值怎么求?

我的回答:
由柯西有
\[f(x)=\frac x2+\frac 12\sqrt {(1+3)\left( {\left( {\frac 12-x} \right)^2+\frac 34} \right)}\geqslant \frac x2+\frac 12\left( {\frac 12-x+\frac 32} \right)=1,\]当 x=0 时取等。

发布于 04-16

注:这题本不必存档,因为太简单,现在存的主要是在我回答下的这个评论:
谢灵        06-09
没证先下了结论:(1/2-x)^2,也是 (x-1/2)^2。你带偏见的强行规定:1/2-x。其实也可以是x-1/2。
所以也可以是:f(x)≥x/2+(1/2)(x-1/2+3/2)=x+1/2
“带偏见的强行规定”随手撸道题都成偏见了我当时真不知道怎么回复他,就没管了。
后来也有人帮我回复,结果:
CuKing        回复谢灵        06-15
什么叫带偏见,两种都对,只不过你这样得不到最小是多少,答主选择了能得到最小值的方法

谢灵        回复CuKing        06-15
f(x)≥x/2+(1/2)(x-1/2+3/2)=x+1/2
x=0,最小值 1/2。

CuKing        回复谢灵        06-15
你看看你x=0,不等式取的是等号吗?

你这么算,我还直接令x=-10000然后得到最小值是-9999.5呢

谢灵        回复CuKing        06-15
f(x)≥x/2+(1/2)(x-1/2+3/2)=x+1/2

x=0,不等式取等号,f(x)=x/2+(1/2)(x-1/2+3/2)=x+1/2
f(x)=0/2+(1/2)(0-1/2+3/2)=0+1/2=1/2

谢灵        回复CuKing        06-15
你这么算,我还直接令x=-10000然后得到最小值是-9999.5呢
===========
这正好证明 他的方法错误。谢谢你了。

CuKing        回复谢灵        06-15
f(x)是题目里带根号的那个,不是x/2+(1/2)(x-1/2+3/2) 答主的思路是首先f(x)一定不小于1,其次令x=0则f(0)=1,所以f(x)最小值就是1。

谢灵        回复CuKing        06-15
f(x)是题目里带根号的那个。
他的方法去根号,错了。
===========

答主的思路是首先f(x)一定不小于1,其次令x=0则f(0)=1,所以f(x)最小值就是1。===== 你这个就错了。

我也可以:答主的思路是首先f(x)一定不小于1,其次令x=0则f(0)=1/2,所以f(x)最小值就是1/2。

CuKing        回复谢灵        06-15
f(0)就是1啊,你咋算的1/2

谢灵        回复CuKing        06-16
f(0)就是1啊,你咋算的1/2
===============
f(x)=x/2+(1/2)(x-1/2+3/2)=x+1/2
x=0,
f(0)=x/2+(1/2)(0-1/2+3/2)=1/2(-1/2+3/2)=1/2(2/2)=1/2

最小值 1/2。

CuKing        回复谢灵        06-16
你这是哪门子f(0)啊,你看题啊,有根号那个
所以我当时没回复也是明智嘀……

TOP

网址:https://www.zhihu.com/question/458756511
标题:这两道题的解法不太自然,大佬们能否解释下这是怎么想到的?
(这里只录我答的那题,由于原图效果差,码一下题目)
(2020莆田期中)在锐角三角形 `ABC` 中,(省略例牌废话),若 `a^2+2ab\cos C=3b^2`,则 `\frac{\tan A}{\tan B\tan C}+\frac6{\tan A}` 的最小值为( )
v2-5a514e7f71fb2d14e2a2830059619e37_r.jpg
2021-7-21 15:36

v2-0f2c6b76070fe34eb8bda8381578821b_r.jpg
2021-7-21 15:36

我的回答:
如果是我的话,在余弦定理变成 `2(a^2-b^2)=c^2` 之后,我就会想到作 c 边上的高 CD(D 为垂足),这样 `a^2-b^2=BD^2-AD^2=(BD-AD)c` ,所以 2(BD-AD) = c = BD+AD,即 BD = 3AD,从而 tanA = 3tanB,下同原解析。

发布于 05-11

TOP

网址:https://www.zhihu.com/question/458990078
标题:这道向量题有什么纯几何或者纯代数解法嘛?
v2-bfb7a02546a3f172eb0030d0219c2f83_r.jpg
2021-7-21 15:46

我的回答:
注意到
\[\bm b\cdot\bm c=\bm b^2+\frac1{16}\bm a^2+\frac 12(2\bm c-\bm a)\cdot(\bm c-\bm b)-\left( \frac14\bm a+\bm b-\bm c \right)^2,\]代入已知条件,即
\[\bm b\cdot\bm c=4+\frac14-\left( \frac14\bm a+\bm b-\bm c \right)^2\leqslant\frac{17}4,\]取等略。

发布于 05-12

注:典型装X解法,常规的代数解法及几何解法见原帖其他回答。

TOP

网址:https://www.zhihu.com/question/459049551
标题:如何证明以下等式,时间2021/05/12 16:17:22?
有三个角,如果任意一个 `A\pm B\pm C` 的角度为 `\pi` 的倍数,证明:
`\sin^4A+\sin^4B+\sin^4C - 2(\sin^2A\sin^2B+\sin^2A\sin^2C+\sin^2B\sin^2C) + 4\sin^2A\sin^2B\sin^2C = 0`

我的回答:
由二倍角公式 `\sin ^2A=\frac {1-\cos 2A}2` 得
\[LHS=\frac 14\sum (1-\cos 2A)^2-\frac 12\sum (1-\cos 2A)(1-\cos 2B)+\frac 12\prod (1-\cos 2A),\]展开化简得
\[LHS=\frac 14(\cos ^22A+\cos ^22B+\cos ^22C-1-2\cos 2A\cos 2B\cos 2C),\]因为
\begin{align*} \cos ^22B+\cos ^22C-1&=\frac {\cos 4B+\cos 4C}2=\cos (2B+2C)\cos (2B-2C),\\ 2\cos 2B\cos 2C&=\cos (2B+2C)+\cos (2B-2C), \end{align*}所以
\begin{align*} LHS&=\frac 14\Bigl( \cos ^22A+\cos (2B+2C)\cos (2B-2C)-\cos 2A\bigl( \cos (2B+2C)+\cos (2B-2C) \bigr) \Bigr)\\ &=\frac 14\bigl( \cos 2A-\cos (2B+2C) \bigr)\bigl( \cos 2A-\cos (2B-2C) \bigr)\\ &=\sin (A+B+C)\sin (A-B-C)\sin (A+B-C)\sin (A-B+C). \end{align*}
发布于 05-12

TOP

网址:https://www.zhihu.com/question/459103027
标题:请问这道题如何做?
v2-2ae0496b3948575ea0706a955c2430fb_r.jpg
2021-7-21 15:55

我的回答:
就是那个经典的爪机解锁题:
v2-c5be5bad48b8ce1617356be9ed52c008_720w.jpg
2021-7-21 15:56

(原帖已经找不到了,只存了这个图)

不断求导有
\begin{align*} F(x)&=\int_0^x{(x^2-t^2)f(t)d t}\\ &=x^2\int_0^x{f(t)d t}-\int_0^x{t^2f(t)d t}, \\ F'(x)&=2x\int_0^x{f(t)d t}+x^2f(x)-x^2f(x)\\ &=2x\int_0^x{f(t)d t}, \\ F''(x)&=2\int_0^x{f(t)d t}-2xf(x), \\ F'''(x)&=2f(x)-2f(x)-2xf'(x)\\ &=-2xf'(x), \end{align*}因为 f'(0)≠0,即 F'''(x) 与 x 同阶,故 F'(x) 与 x^3 同阶,所以 k=3。

发布于 05-13

TOP

网址:https://www.zhihu.com/question/460968964
标题:如何证明这个三角函数等式?
v2-22f2dbaa4fa15322a5ba0db1b14b14f6_r.png
2021-7-21 16:15

我的回答:
【引理】令 `\theta =\frac {2\pi }{13}` 以及 `x=\cos \theta +i\sin \theta ` ,则有
\[1+2(x+x^3+x^4+x^9+x^{10}+x^{12})=\sqrt {13}.\]引理的一般情况参见 二次高斯和 ,不过仅就上式而言直接证明只需平方展开,利用 `x^{13}=1` 可算出左边的平方为 `25+12(x+x^2+\cdots +x^{12})` ,也就是 13。

由引理有 `1+2(\cos \theta +\cos 3\theta +\cos 4\theta +\cos 9\theta +\cos 10\theta +\cos 12\theta )=\sqrt {13}` ,而 `\cos k\theta =\cos (13-k)\theta ` ,所以 `1+4(\cos \theta +\cos 3\theta +\cos 4\theta )=\sqrt {13}` ,即
\[\cos \theta +\cos 3\theta +\cos 4\theta={\frac {-1+\sqrt {13}}4}.\]
回到原题,原式左边就是 `\sin \theta -\sin 4\theta +\sin 3\theta ` ,与上式相近,算它们的平方和,易得
\[(\sin \theta +\sin 3\theta -\sin 4\theta )^2+(\cos \theta +\cos 3\theta +\cos 4\theta )^2=3+2(\cos 2\theta +\cos 5\theta +\cos 7\theta ),\]而 `\cos 7\theta =\cos 6\theta ` ,且易证 `\cos \theta +\cos 2\theta +\cdots +\cos 6\theta =-\frac12` ,所以上式化为
\[(\sin \theta +\sin 3\theta -\sin 4\theta )^2+(\cos \theta +\cos 3\theta +\cos 4\theta )^2=2-2(\cos \theta +\cos 3\theta +\cos 4\theta ),\]所以
\begin{align*} (\sin \theta +\sin 3\theta -\sin 4\theta )^2&=3-(\cos \theta +\cos 3\theta +\cos 4\theta +1)^2\\ &=3-\left( {\frac {-1+\sqrt {13}}4+1} \right)^2\\ &=\frac {13-3\sqrt {13}}8, \end{align*}即得证。

编辑于 05-24

TOP

网址:https://www.zhihu.com/question/463839274
标题:这个关于组合的等式怎么证明呢?
v2-701320cc8b7446de1fddcf532db01328_b.png
2021-7-21 20:57

我的回答:
因为
\begin{align*} \frac{\binom ui\binom vj}{\binom{u+v}{i+j}} &=\frac{\frac{u!}{i!(u-i)!}\cdot\frac{v!}{j!(v-j)!}}{\frac{(u+v)!}{(i+j)!(u+v-i-j)!}}\\ &=\frac{u!v!}{(u+v)!}\cdot\frac{(i+j)!}{i!j!}\cdot\frac{(u-i+v-j)!}{(u-i)!(v-j)!}\\ &=\frac1{\binom{u+v}u}\binom{i+j}i\binom{u-i+v-j}{u-i}, \end{align*}所以待证的等式等价于
\[\sum_{j=0}^v\binom{i+j}i\binom{u-i+v-j}{u-i} =\binom{u+v}u\frac{u+v+1}{u+1}=\binom{u+1+v}{u+1}.\]下面构造路径问题来证明上式,如下图:
v2-d962fff27ebff6fc3b529617dad88543_720w.jpg
2021-7-21 20:57

沿着网格线,只能往上或往右走,易知由 (0,0) 到 `(u+1,v)` 的路径总数为 `\binom{u+1+v}{u+1}` 。

现在,设线段 `l_j` ( `j=0,1,\ldots,v` )的两端分别为 `(i,j), (i+1,j)` ,考虑经过线段 `l_j` 的路径条数,因为由 (0,0) 到 `(i, j)` 有 `\binom{i+j}i` 条,由 `(i+1, j)` 到 `(u+1,v)` 有 `\binom{u-i+v-j}{u-i}` 条,两者之积就是经过 `l_j` 的条数,这样我们就得到了恒等式
\[\sum_{j=0}^v\binom{i+j}i\binom{u-i+v-j}{u-i} =\binom{u+1+v}{u+1}.\]
发布于 06-09

注:其实和四年前的这帖(4#)是等价的,不过看起来这个美观一点。

TOP

网址:https://www.zhihu.com/question/466659554
标题:请问下面这个等式如何证明?
`\sum\limits_{i=k}^{n}{(it-ns)C_{n}^{i}{{s}^{i-1}}{{(t-s)}^{n-i-1}}}=kC_{n}^{k}{{s}^{k-1}}{{(t-s)}^{n-k}}`

我的回答:
先简化一下形式吧,令 `t=(1+a^{-1})s` ,待证等式就可以化为
\[\sum_{i=k}^n\bigl(i-(n-i)a\bigr)C_n^ia^i=kC_n^ka^k,\]然后由
\begin{align*} \bigl(i-(n-i)a\bigr)C_n^ia^i&=iC_n^ia^i-(n-i)C_n^ia^{i+1}\\ &=iC_n^ia^i-(i+1)C_n^{i+1}a^{i+1}, \end{align*}求和即得。

发布于 06-24

TOP

网址:https://www.zhihu.com/question/468984511
标题:如何证明下图所示函数在定义域内恒大于零?
v2-e0e6eeb0e460466013ad141f7d1b31a5_r.jpg
2021-7-21 22:03

(其实是大于等于零)

我的回答:
作置换 `\sqrt x\to x` 后取对数,等价于证明
\[f(x)=\frac {2x^2}{1+x^2}\ln x-\ln (x^2-x+1)\]在定义域内非负。求导整理得
\[f'(x)=\frac {4x}{(1+x^2)^2}\left( {\ln x+\frac {1-x^4}{4x(1-x+x^2)}} \right),\]令
\[g(x)=\ln x+\frac {1-x^4}{4x(1-x+x^2)}\implies g'(x)=-\frac {(1-x)^2(1+x^2)(1-4x+x^2)}{4x^2(1-x+x^2)^2},\]方程 `1-4x+x^2=0` 的两根为 `x=2\pm \sqrt 3` ,所以:

在 `(0,2-\sqrt 3)` 上 `g(x)\searrow ` ,在 `(2-\sqrt 3,2+\sqrt 3)` 上 `g(x)\nearrow ` ,在 `(2+\sqrt 3,+\infty )` 上 `g(x)\searrow ` 。

易证 `g(0^+)=+\infty ` , `g(+\infty )=-\infty` , `g(1)=0` ,故存在 `x_1\in (0,2-\sqrt 3)` , `x_2\in (2+\sqrt 3,+\infty )` 使得:

在 `(0,x_1)` 上 `g(x)>0` ,在 `(x_1,1)` 上 `g(x)<0` ,在 `(1,x_2)` 上 `g(x)>0` ,在 `(x_2,+\infty )` 上 `g(x)<0` 。

所以 f(x) 的单调性是 `\nearrow \searrow \nearrow \searrow ` (中间的极小值是 f(1)),易证 `f(0^+)=f(1)=f(+\infty )=0` ,从而 `f(x)\geqslant 0` 恒成立。

发布于 06-30

TOP

网址:https://www.zhihu.com/question/470289434
标题:如何化简下面这个式子?
v2-a2cbe4a637f3775ea78a315f57ff4de7_r.jpg
2021-7-21 22:29

我的回答:
因为
\begin{align*} \sqrt {10+\sqrt k}-\sqrt {10-\sqrt k}&=\sqrt {20-2\sqrt {10+\sqrt k}\sqrt {10-\sqrt k}}\\ &=\sqrt 2\sqrt {10-\sqrt {100-k}}, \end{align*}所以
\begin{align*} &\text{原式}-1\\ ={}&\frac {\sqrt {10+\sqrt 1}-\sqrt {10-\sqrt 1}+\sqrt {10+\sqrt 2}-\sqrt {10-\sqrt 2}+\cdots +\sqrt {10+\sqrt {99}}-\sqrt {10-\sqrt {99}}}{\sqrt {10-\sqrt 1}+\sqrt {10-\sqrt 2}+\cdots +\sqrt {10-\sqrt {99}}}\\ ={}&\frac {\sqrt 2\sqrt {10-\sqrt {99}}+\sqrt 2\sqrt {10-\sqrt {98}}+\cdots +\sqrt 2\sqrt {10-\sqrt 1}}{\sqrt {10-\sqrt 1}+\sqrt {10-\sqrt 2}+\cdots +\sqrt {10-\sqrt {99}}}\\ ={}&\sqrt 2, \end{align*}即 原式 = `1+\sqrt 2` 。

发布于 07-05

注:录入时发现那边在修改为 LaTeX 格式时分母输入错了,还好可以在问题日志里找回原图。

TOP

网址:https://www.zhihu.com/question/472929869
标题:这个逆是咋求哒?
v2-cecfd6ade0fdba87c33f6286eef7300c_b.jpg
2021-7-21 22:55

我的回答:
\begin{align*} \frac 1{a+b\omega +c\omega ^2}&=\frac 1{a+b\omega +c(-\omega -1)}\\ &=\frac 1{p+q\omega }\qquad(p=a-c,q=b-c)\\ &=\frac {p^2-pq\omega +q^2\omega ^2}{p^3+(q\omega )^3}\\ &=\frac {p^2-pq\omega +q^2\omega ^2}{p^3+q^3}, \end{align*}即纸上的结果。

还可以继续把 `\omega^2` 写成 `-\omega-1` :
\begin{align*} \cdots &=\frac {p^2-q^2-(pq+q^2)\omega }{p^3+q^3}\\ &=\frac {p-q-q\omega }{p^2-pq+q^2}\\ &=\frac {a-b-(b-c)\omega }{(a-c)^2-(a-c)(b-c)+(b-c)^2}\\ &=\frac {a+b\omega ^2+c\omega }{a^2+b^2+c^2-ab-bc-ca}, \end{align*}即与 @Unduloid 给出的公式相同。

编辑于 07-19

注:网友“Unduloid”的回答还给出了更一般的有理化的行列式方法及相关资料(虽然我完全看8懂……

TOP

还有一个私信的回答:

v2-659c7b76c58c714da2e9837c612a7e9b_hd.jpg
2021-7-22 14:05


求证:`2xe^{x-1}\geqslant(x^2+1)(\ln x+1)`。

(1)若 `x\in(0,1]`,有 `e^{x-1}\geqslant x`, `\ln x\leqslant\frac{2(x-1)}{x+1}`,只需证
\[2x^2\geqslant(x^2+1)\left( \frac{2(x-1)}{x+1}+1 \right)\iff\frac{(1-x)^3}{1+x}\geqslant0,\]显然;

(2)若 `x\in(1,+\infty)`,不等式变形为
\[e^{\ln\frac{2x}{x^2+1}+x-1}\geqslant\ln x+1,\]只需证
\[\ln\frac{2x}{x^2+1}+x\geqslant\ln x+1\iff x-1\geqslant\ln\frac{x^2+1}2\iff e^{x-1}\geqslant\frac{x^2+1}2,\]令 `x=1+u`, `u>0`,即证 `e^u\geqslant\frac12u^2+u+1`,也显然。

日期:02-25

TOP

网址:https://www.zhihu.com/question/455929302
标题:这个分式计算的计算结果是如何得到的?
v2-26bad265362d8e190cb4eab9c8b04f50_b.png
2021-7-26 19:15

书上说是用部分分式计算,我没想明白用部分分式怎么算

我的回答:
用拉格朗即可。

设二次函数 `f(x)=(x-p)(x-q)` ,根据拉格朗**值公式,对互不相同的 a, b, c,有
\[f(x)=\frac{(x-b)(x-c)}{(a-b)(a-c)}f(a)+\frac{(x-c)(x-a)}{(b-c)(b-a)}f(b)+\frac{(x-a)(x-b)}{(c-a)(c-b)}f(c),\]两边除以 `(x-a)(x-b)(x-c)` ,再代入 f,即得
\[\frac{(x-p)(x-q)}{(x-a)(x-b)(x-c)}=\frac{(a-p)(a-q)}{(a-b)(a-c)(x-a)}+\frac{(b-p)(b-q)}{(b-c)(b-a)(x-b)}+\frac{(c-p)(c-q)}{(c-a)(c-b)(x-c)}, \]上式作置换 `(x,a,b,c,p,q)\to(\lambda,-a^2,-b^2,-c^2,\mu,\nu)` 就是书上的式子。

类似地可以写出这种形式的 n 元推广,具体就不码了。

发布于 07-25

注:和之前 http://kuing.orzweb.net/viewthread.php?tid=6913 这帖类似。

TOP

网址:https://www.zhihu.com/question/476791407
标题:如何用较为简洁的物理学方程得出结果?
v2-23f2f0d5e15684091eea9cdbc8863adc_r (1).jpg
2021-8-3 15:16
v2-c6b78ab0ad15514917e49afaeff44736_r.jpg
2021-8-3 15:14

本题如果建系建在不好的位置的话式子很难处理,求此题的一般解法?

我的回答:
好多年没撸物理题,希望没记错公式……

我喜欢把它反过来看,即:

从球顶 P 处抛出一物体,在不碰到球体的条件下,求落地瞬间的最小速度。

记初速度及落地速度分别为 `v_0` 及 `v_D` ,则 `v_D^2-v_0^2=4gR` ,所以落地速度仅由初速度决定,那么问题变为:

求从球顶 P 处抛出物体的最小速度,使之不碰到球体。

显然最小时必然抛物线与球相切,如下图:
QQ截图20210803141415.png
2021-8-3 15:14

设在切点 Q 处的速度为 `v_Q` ,由 P 到 Q 的时间为 t,记 `\angle POQ=\alpha` ,则有
\begin{align*}
v_Q\cos\alpha\cdot t&=R\sin\alpha,\\
v_Q\sin\alpha\cdot t-\frac12gt^2&=R-R\cos\alpha,
\end{align*}消 t 化简得
\[v_Q^2=\frac{1+\cos\alpha}{2\cos\alpha}gR,\]于是
\begin{align*}
v_0^2&=v_Q^2-2g(R-R\cos\alpha)\\
&=\frac{1+\cos\alpha}{2\cos\alpha}gR-2(1-\cos\alpha)gR\\
&=\left( \frac1{2\cos\alpha}+2\cos\alpha-\frac32 \right)gR\\
&\geqslant\frac12gR,
\end{align*}当 `\alpha=60^\circ` 时取等,所以
\[v_D^2=v_0^2+4gR\geqslant\frac92gR,\]即原题所求的 `v_{\min}=\sqrt{\frac92gR}` 。

接下来求 d 和 `\theta` ,以 O 为原点建系,由抛物线过 P(0,R)、 `Q(R\cos 30^\circ,R\sin30^\circ)` 以及在 Q 处的导数为 `-\sqrt3` ,不难算出抛物线方程为 `y=-\frac4{3R}x^2+\frac1{\sqrt3}x+R` ,令 y = -R 解得 `d=\frac{\sqrt3+3\sqrt{11}}8R` ,求导再代入这个 d 化简得 `-\sqrt{11}` ,所以 `\theta=\arctan\sqrt{11}` 。

编辑于 08-03

TOP

返回列表 回复 发帖